Number property question from practice test 1

This topic has expert replies
Newbie | Next Rank: 10 Posts
Posts: 8
Joined: Sun Mar 30, 2008 6:43 am
Thanked: 1 times

Number property question from practice test 1

by ksutthi » Sat Jul 05, 2008 11:15 pm
if x is positive, which of the following could be the correct ordering of 1/x, 2x and x^2?

I. x^2 < 2x < 1/x
II. x^2 < 1/x < 2x
III. 2x < x^2 < 1/x

A. None
B. I only
C. III only
D. I and II only
E. I, II and III

Can somebody help me figuring this out? Thanks.

Newbie | Next Rank: 10 Posts
Posts: 3
Joined: Thu Jun 19, 2008 5:22 am
Location: hyderabad

hi

by vsailesh » Sat Jul 05, 2008 11:41 pm
consider x=0.5
1/x=2
2x=1
x^2=0.25
which will give us
x^2<2x<1/x
so b is the answer

Newbie | Next Rank: 10 Posts
Posts: 3
Joined: Thu Jun 19, 2008 5:22 am
Location: hyderabad

hi

by vsailesh » Sat Jul 05, 2008 11:46 pm
consider x=0.5
1/x=2
2x=1
x^2=0.25
which will give us
x^2<2x<1/x
so b is the answer

Newbie | Next Rank: 10 Posts
Posts: 8
Joined: Sun Mar 30, 2008 6:43 am
Thanked: 1 times

by ksutthi » Sun Jul 06, 2008 12:32 am
Thanks. I got the same answer (B), which is wrong.

The answer as indicated in the test is actually (D).

Can anyone help?

GMAT/MBA Expert

User avatar
GMAT Instructor
Posts: 2621
Joined: Mon Jun 02, 2008 3:17 am
Location: Montreal
Thanked: 1090 times
Followed by:355 members
GMAT Score:780

by Ian Stewart » Sun Jul 06, 2008 6:27 am
Thankfully, x is positive here. That means we can divide or multiply both sides of an inequality by x without needing to worry about whether to reverse the inequality. Look at:

II. x^2 < 1/x < 2x

This is really three different inequalities:

a) x^2 < 1/x
b) 1/x < 2x
c) x^2 < 2x

Look at a):

x^2 < 1/x
x^3 < 1
x < 1

So II could only possibly be true for x < 1. We need to check the other conditions though:

Look at b):

1/x < 2x
1 < 2x^2
1/sqrt(2) < x
sqrt(2)/2 < x

So we now see that, for II to be true, x must be between sqrt(2)/2 and 1. c) doesn't tell us anything more. But if you choose any x between sqrt(2)/2 and 1, you'll see how II can hold (try x = 3/4, for example).
For online GMAT math tutoring, or to buy my higher-level Quant books and problem sets, contact me at ianstewartgmat at gmail.com

ianstewartgmat.com

User avatar
Legendary Member
Posts: 2134
Joined: Mon Oct 20, 2008 11:26 pm
Thanked: 237 times
Followed by:25 members
GMAT Score:730
ksutthi wrote:if x is positive, which of the following could be the correct ordering of 1/x, 2x and x^2?

I. x^2 < 2x < 1/x
II. x^2 < 1/x < 2x
III. 2x < x^2 < 1/x

A. None
B. I only
C. III only
D. I and II only
E. I, II and III

Can somebody help me figuring this out? Thanks.
This is a tricky question. We know that X is positive and the answer choices did not place the X^2 as the biggest number so we know that we are dealing with the GMAT ZONE! which is between 0 and 1

I guess it is easy to eliminate the III option. X^2 can be greater than 2x only when X>2 but this will not hold for 1/x>X^2

Between I and II

1/x - 2x > 0

and

2x -1/x > 0

is the trickiest part

and they are both verified.

One take away is: When you see a number before X and you are comparing it with only X - BE CAREFUL

2x vs X
LGTCH
---------------------
"DON'T LET ANYONE STEAL YOUR DREAM!"

User avatar
Legendary Member
Posts: 682
Joined: Fri Jan 16, 2009 2:40 am
Thanked: 32 times
Followed by:1 members

by Vemuri » Sun Apr 26, 2009 10:53 pm
Ian Stewart wrote:Thankfully, x is positive here. That means we can divide or multiply both sides of an inequality by x without needing to worry about whether to reverse the inequality. Look at:

II. x^2 < 1/x < 2x

This is really three different inequalities:

a) x^2 < 1/x
b) 1/x < 2x
c) x^2 < 2x

Look at a):

x^2 < 1/x
x^3 < 1
x < 1

So II could only possibly be true for x < 1. We need to check the other conditions though:

Look at b):

1/x < 2x
1 < 2x^2
1/sqrt(2) < x
sqrt(2)/2 < x

So we now see that, for II to be true, x must be between sqrt(2)/2 and 1. c) doesn't tell us anything more. But if you choose any x between sqrt(2)/2 and 1, you'll see how II can hold (try x = 3/4, for example).
That's a good explanation Ian (as always). I have a question though. From your approach, looks like we have to check for which conditions consistently maintain. The one that gives a contradictory condition is to be eliminated. Is this right?

I. x^2 < 2x < 1/x
The 3 possible conditions are:
a) x^2<2x ==> x<2
b) x^2<1/x ==> x<1
c) 2x<1/x ==> x<1/sqrt(2)
So, x has a value less than 1/sqrt(2)

II. x^2 < 1/x < 2x
The 3 possible conditions are:
a) x^2<1/x ==> x<1
b) 1/x<2x ==> x>1/sqrt(2)
c) x^2<2x ==> x<2
So, x has a value between 1/sqrt(2) & 1

III. 2x < x^2 < 1/x
The 3 possible conditions are:
a) 2x<x^2 ==> x>2
b) 2x<1/x ==> x<1/sqrt(2)
c) x^2<1/x ==> x<1
This does not boil down to a range value for x (its either less than 1 or greater than 2).

I have almost always been unsuccessful with "Must be True" type of questions when plugging in numbers.

GMAT/MBA Expert

User avatar
GMAT Instructor
Posts: 2621
Joined: Mon Jun 02, 2008 3:17 am
Location: Montreal
Thanked: 1090 times
Followed by:355 members
GMAT Score:780

by Ian Stewart » Mon Apr 27, 2009 6:23 am
Vemuri wrote:
That's a good explanation Ian (as always). I have a question though. From your approach, looks like we have to check for which conditions consistently maintain. The one that gives a contradictory condition is to be eliminated. Is this right?

I have almost always been unsuccessful with "Must be True" type of questions when plugging in numbers.
Yes, everything you've done is correct. For a 'could be true' question, like the above, you might be able to make a quick decision about one or more of the items by trying out a simple value for x, just to cut down on some of the algebraic work. So for I) here, you might notice that the inequalities are true for x = 1/2, so I) can be true, and then you only need to check II) and III) algebraically.

Then when you do the algebra here, if you arrive at an answer which is logically impossible (as you did with III) - you found that x was both greater than 2 and less than 1), then the item certainly cannot be true. If, on the other hand, you arrive at a range of possible values for x, as you did with II), as long as you've completely considered all of the implications of the information provided, you will have found all of the values which x can have. And by breaking down the three-part inequality into three two-part inequalities, we can be certain that we are using all of our information here.
For online GMAT math tutoring, or to buy my higher-level Quant books and problem sets, contact me at ianstewartgmat at gmail.com

ianstewartgmat.com

User avatar
Legendary Member
Posts: 682
Joined: Fri Jan 16, 2009 2:40 am
Thanked: 32 times
Followed by:1 members

by Vemuri » Mon Apr 27, 2009 4:18 pm
Thanks a ton Ian !!! Your explanation is crystal clear.

Master | Next Rank: 500 Posts
Posts: 324
Joined: Thu Dec 24, 2009 6:29 am
Thanked: 17 times
Followed by:1 members

by rahul.s » Tue Jan 26, 2010 10:23 am
what's the OA?

Master | Next Rank: 500 Posts
Posts: 301
Joined: Sun Mar 21, 2010 3:18 pm
Thanked: 4 times

by ansumania » Thu Jul 29, 2010 5:23 pm
I got it twice in Gmat prep and got it wrong both the times......

DS is nightmare....:(

User avatar
Community Manager
Posts: 1048
Joined: Mon Aug 17, 2009 3:26 am
Location: India
Thanked: 51 times
Followed by:27 members
GMAT Score:670

by arora007 » Sun Feb 20, 2011 5:28 am
Ian wonderfully explained!
https://www.skiponemeal.org/
https://twitter.com/skiponemeal
Few things are impossible to diligence & skill.Great works are performed not by strength,but by perseverance

pm me if you find junk/spam/abusive language, Lets keep our community clean!!